K
Khách

Hãy nhập câu hỏi của bạn vào đây, nếu là tài khoản VIP, bạn sẽ được ưu tiên trả lời.

3 tháng 9 2018

Đặt \(a=\frac{1}{x},b=\frac{1}{y},c=\frac{1}{z}\),xyz=1  

Cần CM: \(1+\frac{3}{\frac{1}{x}+\frac{1}{y}+\frac{1}{z}}\ge\frac{6}{\frac{1}{xy}+\frac{1}{yz}+\frac{1}{zx}}\) 

\(\Leftrightarrow1+\frac{3}{xy+yz+zx}\ge\frac{6}{x+y+z}\) 

Thật vậy \(1+\frac{3}{xy+yz+zx}\ge1+\frac{9}{\left(x+y+z\right)^2}\ge2\sqrt{\frac{9}{x+y+z}}=\frac{6}{x+y+z}\)(đpcm) 

Dấu "=" xảy ra khi a=b=c=1

20 tháng 8 2017

1.

\(-1\le a\le2\Rightarrow\hept{\begin{cases}a+1\ge0\\a-2\le0\end{cases}\Rightarrow\left(a+1\right)\left(a-2\right)\le0\Leftrightarrow a^2\le}2+a\)

Tương tự \(b^2\le2+b,c^2\le2+c\Rightarrow a^2+b^2+c^2\le6+a+b+c=6\)

Dấu "=" xảy ra khi a=2,b=c=-1 và các hoán vị của chúng

20 tháng 8 2017

Xét \(\frac{a^2+1}{a}=a+\frac{1}{a}\)

Dễ thấy dấu "=" xảy ra khi  \(a=\frac{1}{3}\)

khi đó \(a+\frac{1}{a}=a+\frac{1}{9a}+\frac{8}{9a}\ge2\sqrt{\frac{a.1}{9a}}+\frac{8}{\frac{9.1}{3}}=\frac{10}{3}\)

\(\Rightarrow\frac{a}{a^2+1}\le\frac{3}{10}\)

tương tự =>đpcm

27 tháng 11 2019

k đúng cho mk đi rùi mk giải cho

27 tháng 11 2019

tiếng việt lớp 1 ???????????

4 tháng 7 2018

  Đặt x = 1/a ; y = 1/b, z = 1/c với x,y,z > 0 
đk <=> 1/x + 1/y + 1/z = 1/(xyz) 
<=> xy + yz + zx = 1 
A = √[yz/(1+x²)] + √[zx/(1+y²)] + √[xy/(1+z²)] 
Ta có: 
1 + x² = x² + xy + yz + zx = (x+z)(x+y) 
=> √[yz/(1+x²)] = √[y/(x+y)] . √[z/(x+z)] 
≤ 1/2 . [y/(x+y) + z/(x+z)] (1) 
(áp dụng bđt Cosi: √m .√n ≤ 1/2 . (m+n)) 
Tương tự: 
√[xz/(1+y²)] = √[x/(x+y)] . √[z/(y+z)] ≤ 1/2 . [x/(x+y) + z/(y+z)] (2) 
√[xy/(1+z²)] = √[y/(z+y)] . √[x/(x+z)] ≤ 1/2 . [y/(z+y) + x/(x+z)] (3) 
Cộng vế của (1),(2) và (3) lại ta được: 
A ≤ 1/2 . 3 = 3/2 
Vậy Max A = 3/2 xảy ra <=> x = y = z = 1/√3 <=> a = b = c = √3

7 tháng 7 2018

bạn trả lời lại bằng phần mềm của OLM đươc ko? Thế này hơi khó hiểu bạn ạ! Thanks

1,

\(A=1+a+\frac{1}{b}+\frac{a}{b}+1+b+\frac{1}{a}+\frac{b}{a}\)

\(\ge1+1+2\sqrt{\frac{a}{b}.\frac{b}{a}}+a+b+\frac{a+b}{ab}=4+a+b+\frac{4\left(a+b\right)}{\left(a+b\right)^2}=4+a+b+\frac{4}{a+b}\)

lại có \(\left(1+1\right)\left(a^2+b^2\right)\ge\left(a+b\right)^2\Rightarrow a+b\le\sqrt{2}\)

\(4+a+b+\frac{4}{a+b}=4+\left(a+b+\frac{2}{a+b}\right)+\frac{2}{a+b}\ge4+2\sqrt{2}+\sqrt{2}=4+3\sqrt{2}\)

\(\Rightarrow A\ge4+3\sqrt{2}\)

câu 2

ta có:\(\left(2b^2+a^2\right)\left(2+1\right)\ge\left(2b+a\right)^2\Rightarrow3c\ge a+2b\)

\(\frac{1}{a}+\frac{2}{b}=\frac{1}{a}+\frac{4}{2b}\ge\frac{9}{a+2b}\ge\frac{9}{3c}=\frac{3}{c}\left(Q.E.D\right)\)

14 tháng 12 2019

lớp 1 ????

mà đây cũng đâu phải câu hỏi đâu ??

14 tháng 12 2019

Đây có phải là câu hỏi đâu bạn

28 tháng 12 2017

C.m BĐT phụ \(\frac{a}{b^2+c^2}=\frac{a}{1-a^2}\ge\frac{3\sqrt{3}}{2}a^2\)